LSAT and Law School Admissions Forum

Get expert LSAT preparation and law school admissions advice from PowerScore Test Preparation.

 Steve Stein
PowerScore Staff
  • PowerScore Staff
  • Posts: 1153
  • Joined: Apr 11, 2011
|
#6902
Hey Ray,

That all looks right to me--nice work!

~Steve
 RayMiller
  • Posts: 33
  • Joined: Jul 16, 2012
|
#6904
Oh cool!!

Thanks I guess now I just have to work on trusting myself.

Thanks again
 est15
  • Posts: 94
  • Joined: Aug 28, 2013
|
#15930
I got this one right but I wasn't very sure about my answer so I wanted to make sure I completely understood it.

I diagrammed the argument as:

economy weak :arrow: prices constant
economy weak :arrow: unemployment rises :arrow: investment decreases

(C) must be true because of the contrapositive of the second diagram.

My reason for eliminating (B) was that the passage never states a direct relationship between unemployment and prices, so (B) could be true. Is that correct?

Then for the either/or statements, I followed the diagramming either/or strategy in the Logical Reasoning Bible. The stuff in parentheses are what I determined the statement's validity to be based on the argument I wrote on the very top.

For (A):
~economy weak :arrow: investment decreases (could be true)
~investment decreases :arrow: economy weak (must be false)

For (D):
~economy weak :arrow: prices constant (could be true)
~prices constant :arrow: ~economy weak (must be true)

For (E):
~unemployment rises :arrow: ~economy weak (must be true)
economy weak :arrow: unemployment rises (must be true)

So is (A) the answer because (D) and (E) all have at least one must be true? And does that mean for an either/or you just need at least one of the two diagrams to be must be true?

Thank you!
 Steve Stein
PowerScore Staff
  • PowerScore Staff
  • Posts: 1153
  • Joined: Apr 11, 2011
|
#15931
Hi,

Your diagram looks good; let's also consider the last sentence in the stimulus: Investment is not decreasing. In that case, what else do we know?

Investment decreasing :arrow: unemployment rises :arrow:economy weak

The question asks for the answer choice that Must be False, which means that the right answer will be the one that Cannot be True, while all four incorrect answer choices are possible.

Correct answer choice (A) claims that either the economy is weak or investment is decreasing. Based on the information from the stimulus and highlighted in the diagram above, though, we know that the economy is not weak, and we know that investment is not decreasing. Since this is the only answer that is provably false, it must be the correct answer choice.

Since none of the other answer choices are inconsistent with your diagram of the conditional relationship, or the above diagram, they all could be true, and thus can be ruled out in response to the Cannot Be True question.

I hope that's helpful! Please let me know whether this is clear--thanks!

~Steve
 est15
  • Posts: 94
  • Joined: Aug 28, 2013
|
#15936
Hi Steve,

That makes sense. Thank you.
 Steve Stein
PowerScore Staff
  • PowerScore Staff
  • Posts: 1153
  • Joined: Apr 11, 2011
|
#16125
Hi,

Thank you! I appreciate your response and am glad that you found that helpful!

~Steve
 allisonellen7
  • Posts: 32
  • Joined: Sep 13, 2014
|
#17045
Hi there! I can not understand why D does not have to be false. "Either the economy is weak or prices are remaining constant" must be false because if the economy is weak, prices are remaining constant according to the prompt. Just like the correct answer, A, "either the economy is weak or investment is decreasing" must be false because if the economy is weak, investment must be decreasing. Thanks in advance!
 Nicholas Bruno
PowerScore Staff
  • PowerScore Staff
  • Posts: 62
  • Joined: Sep 27, 2011
|
#17049
Hi,

So from your question, I think you understand why A is the right answer :). So I will just address D.

The key to D is right in the first part of the stimulus: "If the economy is weak, then prices remain constant." This is a sufficient/necessary statement:

Economy is weak :arrow: prices constant

The contrapositive:

If prices are not constant :dblline: economy is not weak.

D in incorrect because it is a mistaken negation--it does not include the "not" that is necessary.

In other words, there can be a situation where the economy is strong AND prices are constant. The conditional relationship does not preclude that.

I hope that this helps!
 temiolof
  • Posts: 8
  • Joined: Aug 30, 2016
|
#34204
Hi PowerScore Staff,

I got this question wrong because I didn't understand how to incorporate "although" [unemployment rises] into the conditional diagram. For some reason, I assumed it meant something like "regardless of if"/"whether or not". It seems clear here that it should be diagrammed as a an additional necessary condition in addition to "prices remain constant". Once I understood this, I could see why A was right and the other answers were wrong.

To clarify, will it always be the case that "although" (along with "even if/though") is an additional necessary condition when introduced within a conditional statement?

Thank you!
 Emily Haney-Caron
PowerScore Staff
  • PowerScore Staff
  • Posts: 577
  • Joined: Jan 12, 2012
|
#34223
Hi temiolof,

Great job really thinking this one through and figuring out where you went wrong! I'd think of this less as an absolute rule for how to approach these questions, and instead focus more on really thinking through the meaning of each question you approach. Does that make sense?

Get the most out of your LSAT Prep Plus subscription.

Analyze and track your performance with our Testing and Analytics Package.